Which of the following expressions is equivalent to the expression given below.

12 (-31 + 26) + 45
—————————
4

3.5

-3.75

4.2

-4.5

Answers

Answer 1

Answer:

wuhh

Step-by-step explanation:

Answer 2
The Answer: -15
I hope this helps you out

Related Questions

An athlete runs 8 miles in 50 minutes on a treadmill at this rate how long will it take the athlete to run 9 miles​

Answers

First you would find out how many minutes it took him to run one mile so it would be 50/8= 6.25 so add one more mile it would be 56 minutes and 15 seconds

Answer:

56.25 minutes

Step-by-step explanation:

1 mile is 6.25 minutes

9 miles is _____?

6.25 minutes x 9 miles =

56.25 minutes

Find Unit Rate to help solve

Hope this helps :)

Write the expanded form of the expression

r(−3−5t)

Answers

-3r -5tr, all you need to do is distribute the variable.

Given that x² -x -2 is a factor of x³ +3 x² +ax +b, calculate the values of a and b and hence find the remaining factor.

Answers

Answer:

[tex]a = -6, b = - 8[/tex]

Remaining factor = [tex]x+4[/tex]

Step-by-step explanation:

Given that:

Two polynomials:

[tex]x^{2} -x-2[/tex] and [tex]x^3 +3 x^2 +ax +b[/tex]

[tex]x^{2} -x-2[/tex] is a factor of [tex]x^3 +3 x^2 +ax +b[/tex].

To find:

The values of [tex]a[/tex] and [tex]b[/tex] from the cubic equation and the remaining factor.

Solution:

Let us first of all, factorize [tex]x^{2} -x-2[/tex].

[tex]x^{2} -2x+x-2=0\\\Rightarrow x(x-2)+1(x-2)=0\\\Rightarrow (x+1)(x-2)=0\\\Rightarrow x = -1, 2[/tex]

There are two factors of the given quadratic equation.

These two factors must also be factors of the cubic equation as well.

Putting [tex]x = 2[/tex]:

[tex]2^3 +3 \times 2^2 +a\times 2 +b =0\\\Rightarrow 2a+b=-20 ..... (1)[/tex]

Putting [tex]x = -1[/tex]:

[tex](-1)^3 +3 \times (-1)^2 +a\times (-1) +b =0\\\Rightarrow -a+b=-2 ..... (2)[/tex]

Subtracting (2) from (1):

[tex]3a = -18\\\Rightarrow a = -6[/tex]

From equation (1):

[tex]2\times (-6) + b=-20\\\Rightarrow b = -8[/tex]

Putting the values of [tex]a[/tex] and [tex]b[/tex] in the cubic equation, we get:

[tex]x^3 +3 x^2 -6x -8[/tex]

Dividing the cubic equation with quadratic, we get:

[tex]\dfrac{x^3 +3 x^2 -6x -8}{x^{2} -x-2} = x+4[/tex]

write in slope-intercept form 3x-9=7y+8

Answers

Answer:

y = 3 /7 x − 17 /7

Step-by-step explanation:

Which number is a solution of the inequality? 8

Answers

Answer:

i need more info or i cannot help

Answer: -1

Step-by-step explanation:Solve: 8(7−x)<64.Divide both sides by 8.7−x<6487−x<8Subtract 7 from both sides.−x<8−7−x<1Multiply both sides by −1. This will reverse the inequality.x>−1

z=32+41.9i
What are the real and imaginary parts of z?
--------------------------------------------------------------
A. Re(z) = 32 and Im(z) = 41.9
B. Re(z) = 32 and Im(z) = 41.9i
C. Re(z) = 41.9i and Im(z) = 32
D. Re(z) = 41.9 and Im(z) = 32

Answers

A. Re(z)= 32 and Im(z)= 41.9

The real part of a complex number z=a+bi is ‘a’, and the imaginary part is ‘b’

a contracter charges 1,200 for 1,100 square feet installed? It costs$ to have 1,100 feet

Answers

Answer:

$1,200

Step-by-step explanation:

i put $100 in a savings account at v% yearly interest. How much money will i have in my account at the end of the year

Answers

Complete question :

i put $100 in a savings account at v% yearly interest. How much money will i have in my account at the end of the year

Answer:

100 + v

Step-by-step explanation:

Given that:

Interest yearly = v%

Amount deposited = principal = $100

Amount earned at the end of the year:

A = P(1 + rt)

A = 100(1 + 0.01v)

A = 100 + v

Where A = final amount

Final amount at the end of the year = 100 + v

please i need this done in an hour

Answers

You need to compare the size to your ruler then you cans find the answer

I need help answering that I’m lost and it’s a 3 step problem

Answers

hi hi hi i believe 32

Which ordered pair does NOT represent a point located in IV Quadrant ?

A (0.001, -0.001)
B (0, -5)
C (0.5, -8.75)
D (9, 999, -9, 999)

Answers

Answer:

B

Step-by-step explanation:

If a point is on the line of origin it doesn't count as being in any quadrant.

SOMEONE HELP PLZ!!!!!! ​

Answers

Answer:b

Step-by-step explanation:

1. Sherrys age is nine less than six times Fred's age. Write an equation and find Sherrys age if Fred is ten years old.

2. A bicycle was originally priced at $102 and is on sale for 30% off. What is the discounted price?

Answers

Answer:

1. x= 10x6-9 Sherry is 51 apparently

2. $71.40

Step-by-step explanation:

Math question can you please help and can you breakdown how you got your answer

Answers

Answer:

8

Step-by-step explanation:

AB=DC

BC=AD

BA=CD

CB=DA

What is the value of the expression below?

Answers

Answer:

-19

General Formulas and Concepts:

Pre-Algebra

Order of Operations: BPEMDAS

Brackets Parenthesis Exponents Multiplication Division Addition Subtraction Left to Right

Step-by-step explanation:

Step 1: Define Expression

3² + (9 - 16)(4)

Step 2: Evaluate

Exponents:                              9 + (9 - 16)(4)(Parenthesis) Subtract:           9 + (-7)(4)Multiply:                                   9 - 28Subtract:                                  -19

Answer:

Step-by-step explanation:

BODMAS=bracket

O=of

D=division

M=multiplication

A=addition

S=subtraction

3*3=9

9+(9-16)(4)

9+(7)(4)

9-28

-19

Which point below is a solution to the equation y = 2x + 3?
A (145, 291)
B (78, 163)
C (37, 77)

Answers

Answer:

C (37, 77)

Step-by-step explanation:

Option A:

[tex]291=2(145)+3\\\\291=290+3\\\\\boxed{291\neq 293}[/tex]

Option A is not a solution.

Option B:

[tex]163=2(78)+3\\\\163=156+3\\\\\boxed{163\neq 159}[/tex]

Option B is not a solution.

Option C:

[tex]77=2(37)+3\\\\77=74+3\\\\\boxed{77=77}[/tex]

Option C is a solution.

Hope this helps.

Answer:

C

Step-by-step explanation:

To determine which point is a solution, substitute the x- coordinate into the equation and if the value is equal to the y- coordinate then it is a solution

A (145, 291 )

y = 2(145) + 3 = 290 + 3 = 293 ≠ 291 ← not a solution

B (78, 163 )

y = 2(78) + 3 = 156 + 3 = 159 ≠ 163 ← not a solution

C (37, 77 )

y = 2(37) + 3 = 74 + 3 = 77 ← solution to the equation

HELP!! (20-30 points) and brainlist

Answers

Answer:

82.8

Step-by-step explanation:

Formula

multiply the length value by 36

A feather fell from a bird's nest in a tree from a height of 25 feet. The leaf fell at a speed of 3 feet per second. Choose all the equations below that correctly describes the relationship between t, the time in seconds, and h, the feathers height in feet?
A) h=25-3t
B) h=3t-25
C) h=-3t-25
D) h+3t=25

Answers

Answer:

A and C

Step-by-step explanation:

Please help and give details :)

Answers

7x+4y-28=0 is the answer

(29-(-7):
= =a
Look at picture

Answers

Answer:

Four

Step-by-step explanation:

29 plus 7 = 36 divided by 4= 9

Simplify the Expression(20 points)-
-m - 7m

Answers

Answer:

-8m

Step-by-step explanation:

Combine like terms

7

{\color{#c92786}{-m}}{\color{#c92786}{-7m}}

−m−7m

8

{\color{#c92786}{-8m}}

−8m

A jar contains 42 red marbles numbered 1 to 42 and 36 blue marbles numbered 1 to 36. A marble is drawn at random from the jar. Find the probability of the given event. Please enter reduced fractions. (a) The marble is red. P(red)= Correct (b) The marble is odd-numbered. P(odd)= Correct (c) The marble is red or odd-numbered. P(red or odd) = Incorrect (d) The marble is blue or even-numbered. P(blue or even) =

Answers

Answer: See explanation

Step-by-step explanation:

Number of red marbles = 42

Number of blue marbles = 36

Total number of marbles = 42 + 36 = 78

There are 21 odd numbers between 1 - 42.

There are 18 odd numbers between 1 - 36.

(a) Probability that the marble is red will be:

P(red)= 42 / 78 = 7 / 13

(b) Probability that the marble is odd-numbered will be:

= 39 / 78 = 1/2

(c) Probability that the marble is red or odd-numbered will be:

P(red or odd) = P(red) + p(odd) - P(odd and red)

= (42/78) + (39/78) - (21/78)

= 60/78

= 10/13

(d) Probability that the marble is blue or even-numbered.

= P(blue) + P(even) - P(blue and even)

= 36/78 + 39/78 - 18/78

= 57/78

= 19/26

What is 2x2 plus 3 times y and it equals b?

Answers

Answer: The answer is 7-b=-y

Step-by-step explanation: You subtract b from one side to the other, and then do the same with y

Samantha created part of a table to show her savings. She saved at the same rate each week. Which graph shows this relationship?

Answers

Answer:

The answer is B

Step-by-step explanation:

because it is going up by 6 so graph B is the only graph going up by 6.

The  correct graph is with $6, $12, $18, $24.

What is Graph?

The graph is simply a structured representation of the data. It aids in our comprehension of the data. The numerical information gathered through observation is referred to as data.

In a line graph, the information or data is represented as a series of markers, or dots, and is then connected to one another by a straight line.

Usually, data that varies over time is represented with a line graph.

Given:

As, from the given graph

The saving increases each week at the same rate therefore saving increases from 3rd to 4th week is 6$. this means that saving increases $6 each week.

Hence, option to is correct with $6, $12, $18, $24.

Learn more about graph here:

https://brainly.com/question/17267403

#SPJ6

what is
4.1 - 1.25 over 0.005


Answers

Answer:

Step-by-step explanation:

4.1-1.25=2.85

lets divide

2.85/0.005=570

Answer:

2.85/0.005=570

Step-by-step explanation:

Hope this helped have an amazing day!

a force of 12N acts for 5 s on a mass of 2 kg. What is the change in momentum of a mass of 10kg under the same condition​

Answers

Answer:

The change in momentum is: 60 N.s

Step-by-step explanation:

By Newton's second law:

F = ∆p / ∆t

Given that

∆t = 5sF = 12N

Using the formula to determine the change in momentum

F = ∆p / ∆t

∆p = F × ∆t

     = 12 × 5

      = 60 N.s

Therefore, the change in momentum is: 60 N.s

George earns commission as a percentage of his tuxedo sales. On Tuesday, he sold $750 worth of tuxedos and earned $60
commission. What is George's commission rate?

Answers

Answer:

8%

Step-by-step explanation:

1% of 750 is 7.50

So 60 divided by 7.5 is 8

Peter is calculating the interest earned on a
deposit of $275 in an account that earns 8%
simple interest after 12 years.

Answers

Answer:

a) He substituted 0.8 instead of 0.08.

b) $264

Step-by-step explanation:

The full question is: a. What did Peter do incorrectly? b. What is the correct amount of interest?

Using the equation [tex]I=prt[/tex], we can substitute the given values.I = 275 x 0.08 x 12I = 264

What is 5[cos(Pi/4) + I sin (pi/4)] raised to the 3rd power?

Answers

Answer: The answer is C on edge

Step-by-step explanation:

Find the unit rate
a. 5 movie tickets for $25.00
b. mowed 5 yards for $35.00
c. 6 calculators cost $240.00
d. 8 dollars for 2 cans of tuna ​

Answers

Answer:

A.  $5

B.  $7

C.  $40

D.  $4

Step-by-step explanation:

To find the unit rate, you need to divide the cost by the amount.

A.  $25/5 = $5 for a movie ticket

B.  $35/5 = $7 for one yard

C.  $240/6 = $40 for a calculator

D.  $8/2 = $4 for a can of tuna

Other Questions
Which statement is the most accurate regarding the meaning of race? A) It is entirely dependent on fixed biological characteristics. B) Race varies depending on the society in which its meaning is constructed. C) The concept of race tends to be quite similar in all cultures. D) Race is a variable that changes depending on the age of the people who use the term. E) All of the above. 0.450 moles of oxygen gas is subjected to a pressure of 3.00 atm and 75 oC.What is it's volume (in Liters)? Which element in Group 18 of the Periodic Table has the highest lonization energy? Fill in the blank. Remember to spell correctly. The Viking who discovered Greenland and started a settlement there was named _______ the Red. Do hawks eat spiders? In the panel discussion, the professor spoke loftily about his role in theresearch. Would you say the professor thought his role was important?Explain. Write a paragraph about volleyball and favourite volleyball sportman (minimum 200 words) please help me learning about the holocaust Number 1-4 plsssss please help me!!!!! Which of the following is true about similar figures?All similar figures are congruent.If two figures are not similar, they can still be congruent.All congruent figures are similar.If two figures are not congruent, they can't be similar. A meal plan would need to accommodate all of the following EXCEPTA. how much exercise you get during the week.B. your food budget for the week or month.C. days that you are unlikely to be eating at home.D. the amount of time that you have to cook on various days. how does the structure help the Hemoglobin perform its function In Act I of The Crucible, why does Reverend Parris seek help for Betty from Reverend Hale? what are the importance of informal education? Based on the context, what meaning is Rachael conveying by using a pun?h"I'm a huge fan of the New York Mets," said Keith."You'd better get down to the field then," said Rachael. "They look likethey're getting overheated."O A. Rachael jokes that Keith should try out as a player.O B. Rachael jokes that Keith's job is to cool the players off.C. Rachael jokes that Keith has an icy personality.D. Rachael jokes that Keith is not a very committed fan. The cost, c(x), for a taxi ride is given by c(x) = 3x + 2.00, where x is thenumber of minutes.On a piece of paper, graph c(x) = 3x + 2.00. Then determine which answer matches the graph you drew, including the correct axis labels, Decide whether each of the following groups of words is a complete sentence (an independent clause) or an incomplete sentence (a dependent clause). If the sentence is incomplete, capitalize it and do whatever else is necessary to make it complete. If the sentence is complete, capitalize the first word and add the appropriate ending punctuation.three ships went with Columbusbecause they found no gold in that countryafter they went running throughhe wants to visit the moonthe boy who has lots of frecklesshe turned a page in her booka book that I readbefore the race began a TV set is pushed a distance of 2 m with a force of 20 N how much work is done on the set $10,000 down payment for your house if you can invest 2% of your monthly income (yearly income: $67,705) into an account that earns 4% quarterly. Also what is the formula for this? Is it compounded quarterly? the the slope and Y-intercept of -4x-2y=-8